Những câu hỏi liên quan
Lê Tài Bảo Châu
Xem chi tiết
Kiệt Nguyễn
1 tháng 8 2020 lúc 8:33

Xét \(\frac{a^3}{a^2+ab+b^2}-\frac{b^3}{a^2+ab+b^2}=\frac{\left(a-b\right)\left(a^2+ab+b^2\right)}{a^2+ab+b^2}=a-b\)

Tương tự, ta được: \(\frac{b^3}{b^2+bc+c^2}-\frac{c^3}{b^2+bc+c^2}=b-c\)\(\frac{c^3}{c^2+ca+a^2}-\frac{a^3}{c^2+ca+a^2}=c-a\)

Cộng theo vế của 3 đẳng thức trên, ta được: \(\left(\frac{a^3}{a^2+ab+b^2}+\frac{b^3}{b^2+bc+c^2}+\frac{c^3}{c^2+ca+a^2}\right)\)\(-\left(\frac{b^3}{a^2+ab+b^2}+\frac{c^3}{b^2+bc+c^2}+\frac{a^3}{c^2+ca+a^2}\right)=0\)

\(\Rightarrow\frac{a^3}{a^2+ab+b^2}+\frac{b^3}{b^2+bc+c^2}+\frac{c^3}{c^2+ca+a^2}\)\(=\frac{b^3}{a^2+ab+b^2}+\frac{c^3}{b^2+bc+c^2}+\frac{a^3}{c^2+ca+a^2}\)

Ta đi chứng minh BĐT phụ sau: \(a^2-ab+b^2\ge\frac{1}{3}\left(a^2+ab+b^2\right)\)(*)

Thật vậy: (*)\(\Leftrightarrow\frac{2}{3}\left(a-b\right)^2\ge0\)*đúng*

\(\Rightarrow2LHS=\Sigma_{cyc}\frac{a^3+b^3}{a^2+ab+b^2}=\Sigma_{cyc}\text{ }\frac{\left(a+b\right)\left(a^2-ab+b^2\right)}{a^2+ab+b^2}\)\(\ge\Sigma_{cyc}\text{ }\frac{\frac{1}{3}\left(a+b\right)\left(a^2+ab+b^2\right)}{a^2+ab+b^2}=\frac{1}{3}\text{​​}\Sigma_{cyc}\left[\left(a+b\right)\right]=\frac{2\left(a+b+c\right)}{3}\)

\(\Rightarrow LHS\ge\frac{a+b+c}{3}=RHS\)(Q.E.D)

Đẳng thức xảy ra khi a = b = c

P/S: Có thể dùng BĐT phụ ở câu 3a để chứng minhxD:

Bình luận (0)
 Khách vãng lai đã xóa
Phùng Minh Quân
27 tháng 7 2020 lúc 22:28

1) ta chứng minh được \(\Sigma\frac{a^4}{\left(a+b\right)\left(a^2+b^2\right)}=\Sigma\frac{b^4}{\left(a+b\right)\left(a^2+b^2\right)}\)

\(VT=\frac{1}{2}\Sigma\frac{a^4+b^4}{\left(a+b\right)\left(a^2+b^2\right)}\ge\frac{1}{4}\Sigma\frac{a^2+b^2}{a+b}\ge\frac{1}{8}\Sigma\left(a+b\right)=\frac{a+b+c+d}{4}\)

bài 2 xem có ghi nhầm ko

Bình luận (0)
 Khách vãng lai đã xóa
Phùng Minh Quân
27 tháng 7 2020 lúc 22:50

3a biến đổi tí là xong

b tuong tự bài 1 

Bình luận (0)
 Khách vãng lai đã xóa
Bùi Nhâm Tú
Xem chi tiết
Kamka Lanka
Xem chi tiết
Thiên An Nguyễn
Xem chi tiết
coldly queen
24 tháng 3 2019 lúc 13:06

​​fddfssdfdsfdssssssssssssssffffffffffffffffffsssssssssssssssssssfsssssssssssssssssssssssfffffffffffffff
Bình luận (0)
bin
24 tháng 3 2019 lúc 13:11

Ez lắm =)

Bài 1:

Với mọi gt \(x,y\in Q\) ta luôn có: 

\(x\le\left|x\right|\) và \(-x\le\left|x\right|\) 

\(y\le\left|y\right|\) và \(-y\le\left|y\right|\Rightarrow x+y\le\left|x\right|+\left|y\right|\) và \(-x-y\le\left|x\right|+\left|y\right|\)

Hay: \(x+y\ge-\left(\left|x\right|+\left|y\right|\right)\)

Do đó: \(-\left(\left|x\right|+\left|y\right|\right)\le x+y\le\left|x\right|+\left|y\right|\)

Vậy: \(\left|x+y\right|\le\left|x\right|+\left|y\right|\)

Dấu "=" xảy ra khi: \(xy\ge0\)

Bình luận (0)
bin
24 tháng 3 2019 lúc 13:22

Bài 3: 

Ta có: \(\frac{x}{a}=\frac{y}{b}=\frac{z}{c}=\frac{x+y+z}{a+b+c}=x+y+z\) (vì a + b + c = 1)

Do đó: \(\left(x+y+z\right)^2=\frac{x^2}{a^2}=\frac{y^2}{b^2}=\frac{z^2}{c^2}=\frac{x^2+y^2+z^2}{a^2+b^2+c^2}=x^2+y^2+z^2\) (vì a2 + b2 + c2 = 1)

Vậy: (x + y + z)2 = x2 + y2 + z2

Bình luận (0)
Wang Jun Kai
Xem chi tiết
hung
Xem chi tiết
Nhật Kim Anh
10 tháng 8 2017 lúc 14:43

hi kết bạn nha

Bình luận (0)
nguyen yen vi
Xem chi tiết
Quyết Tâm Chiến Thắng
Xem chi tiết
zZz Cool Kid_new zZz
30 tháng 8 2019 lúc 19:25

Đặt \(\left(\frac{a-b}{c},\frac{b-c}{a},\frac{c-a}{b}\right)\rightarrow\left(x,y,z\right)\)

Khi đó:\(\left(\frac{c}{a-b},\frac{a}{b-c},\frac{b}{c-a}\right)\rightarrow\left(\frac{1}{x},\frac{1}{y},\frac{1}{z}\right)\)

Ta có:

\(P\cdot Q=\left(x+y+z\right)\left(\frac{1}{x}+\frac{1}{y}+\frac{1}{z}\right)=3+\frac{y+z}{x}+\frac{z+x}{y}+\frac{x+y}{z}\)

Mặt khác:\(\frac{y+z}{x}=\left(\frac{b-c}{a}+\frac{c-a}{b}\right)\cdot\frac{c}{a-b}=\frac{b^2-bc+ac-a^2}{ab}\cdot\frac{c}{a-b}\)

\(=\frac{c\left(a-b\right)\left(c-a-b\right)}{ab\left(a-b\right)}=\frac{c\left(c-a-b\right)}{ab}=\frac{2c^2}{ab}\left(1\right)\)

Tương tự:\(\frac{x+z}{y}=\frac{2a^2}{bc}\left(2\right)\)

\(=\frac{x+y}{z}=\frac{2b^2}{ac}\left(3\right)\)

Từ ( 1 );( 2 );( 3 ) ta có:
\(P\cdot Q=3+\frac{2c^2}{ab}+\frac{2a^2}{bc}+\frac{2b^2}{ac}=3+\frac{2}{abc}\left(a^3+b^3+c^3\right)\)

Ta có:\(a+b+c=0\)

\(\Rightarrow\left(a+b\right)^3=-c^3\)

\(\Rightarrow a^3+b^3+3ab\left(a+b\right)=-c^3\)

\(\Rightarrow a^3+b^3+c^3=3abc\)

Khi đó:\(P\cdot Q=3+\frac{2}{abc}\cdot3abc=9\)

Bình luận (0)
Quyết Tâm Chiến Thắng
30 tháng 8 2019 lúc 19:41

Mách mk nốt 2 bài kia vs

Bình luận (0)
Nguyễn Mạnh Kiên
31 tháng 8 2019 lúc 14:18

chiju

Bình luận (0)
bach nhac lam
Xem chi tiết
Trần Thùy Linh
25 tháng 4 2020 lúc 13:04

\(A=\frac{a}{ab+c\left(a+b+c\right)}+\frac{b}{bc+a\left(a+b+c\right)}+\frac{c}{ca+b\left(a+b+c\right)}\)

\(=\frac{a}{\left(b+c\right)\left(a+c\right)}+\frac{b}{\left(a+b\right)\left(a+c\right)}+\frac{c}{\left(a+b\right)\left(c+b\right)}\)

Áp dụng bđt AM-GM ta có

\(A=\frac{a\left(a+b\right)+b\left(b+c\right)+c\left(c+a\right)}{\left(a+b\right)\left(b+c\right)\left(c+a\right)}\)

\(\ge27.\frac{a^2+b^2+c^2+ab+bc+ca}{8\left(a+b+c\right)^3}\)\(=\frac{a^2+b^2+c^2+ab+bc+ca}{8}\)

\(=\frac{\left(a+b+c\right)^2-\left(ab+bc+ca\right)}{8}\)\(\ge\frac{9-\frac{\left(a+b+c\right)^2}{3}}{8}=\frac{9-3}{8}=\frac{3}{4}\)

Dấu "=" xảy ra khi a=b=c=1

Bình luận (0)
tthnew
25 tháng 4 2020 lúc 16:02

b) Mạnh hơn, và dễ dàng hơn là:

\(\left(\frac{a}{b}+\frac{b}{c}+\frac{c}{a}\right)^2\ge\left(a+b+c\right)\left(\frac{1}{a}+\frac{1}{b}+\frac{1}{c}\right)+\frac{\sum c\left(a-b\right)^2}{abc}\)

Nó tương đương với: \({\frac {{a}^{2}}{{b}^{2}}}+{\frac {{b}^{2}}{{c}^{2}}}+{\frac {{c}^{2} }{{a}^{2}}}+3-2\,{\frac {a}{b}}-2\,{\frac {b}{c}}-2\,{\frac {c}{a}} \geqq 0\)

Là hiển nhiên vì \(\frac{a^2}{b^2}+1\ge\frac{2a}{b}\)

Đơn giản:))

Bình luận (0)
tthnew
25 tháng 4 2020 lúc 16:46

a) Đặt \(\left(x;y;z\right)=\left(\frac{1}{a};\frac{1}{b};\frac{1}{c}\right)\rightarrow ab+bc+ca=1;0< a,b,c< 1\)

Cần chứng minh: \(P=\sum\frac{\frac{1}{a}-1}{\frac{1}{b^2}}=\sum\frac{b^2-ab^2}{a}\ge\sqrt{3}-1\)

Hay là: \(\left(\frac{b^2}{a}+\frac{c^2}{b}+\frac{a^2}{c}\right)\sqrt{ab+bc+ca}\ge\left(\sqrt{3}-1\right)\left(ab+bc+ca\right)+a^2+b^2+c^2\)

\(\Leftrightarrow\left(\frac{b^2}{a}+\frac{c^2}{b}+\frac{a^2}{c}\right)^2\left(ab+bc+ca\right)\ge\) \(\Big[ (\sqrt{3} -1) (ab+bc+ca) +a^2+b^2+c^2\Big]^2\)

Giả sử \(c=\min\{a,b,c\}\) và đặt \(a=c+u, \, b=c+v \, (u,\, v \geq 0)\)

Nếu mình không nhìn nhầm, sau khi rút gọn, nhóm lại theo biến c, bạn nhận được một cái gì đó gọi là hiển nhiên haha

Chúc may mắn, mình mới rút gọn thử thì thấy có vẻ hiển nhiên thật :))

Bình luận (0)